On the norm of a quotient of a Banach space.












38












$begingroup$


Let $E$ be a Banach space and $F$ a closed subspace. It is well known that the quotient space $E/F$ is also a Banach space with respect to the norm
$$
leftVert x+FrightVert_{E/F}=inf{leftVert yrightVert_Emid yin x+F}.
$$
Unfortunately in a set of lecture notes on (Lie) group representations (material for our study group) the author accidentally used here $min$ instead of $inf$. Probably a mostly harmless booboo, because at that point it was only needed to get a Banach space structure on the quotient, and we will probably be concentrating on Hilbert spaces anyway, where the problem does not arise.



Namely from Rudin's Functional Analysis I could not find a proof that the minimum should always be attained. Except in the case of a Hilbert space, where an application of parallelogram law (the sum of the squared norms of the two diagonals of a parallelogram equals that of the four sides) allows us to find a Cauchy sequence among a sequence of vectors $(y_n)subset x+F$ such that
$$lim_{ntoinfty}leftVert y_nrightVert_E=leftVert x+FrightVert_{E/F}.$$



But anyway, the suspicion was left that the infimum is there for a reason (other than conveniently allowing us to sweep this detail under the rug at that point of the development of theory), so in the interest of serving our study group I had to come up with a specific example, where the minimum is not achieved. It's been 25 years since I really had to exercise the Banach space gland in my brain, so it has shrunk to size of a raisin. Searching this site did help, because I found this question. There we have $E=C([0,1])$, the space of continuous real functions on $[0,1]$ equipped with the sup-norm. If we denote by $Lambda$ the continuous functional
$$
Lambda: Etomathbb{R},fmapstoint_0^{1/2}f-int_{1/2}^1f
$$
and let $F=kerLambda$, then the answer to the linked question proves that there is no minimum sup-norm function in the coset $Lambda^{-1}(1)$.



So I have a (counter)example, and the main question has evolved to:




When can we use minimum in place of infimum in the definition of the quotient space norm?




My thinking:




  1. It seems to me that the answer is affirmative, if $F$ has a complement, i.e. we can write $E=Foplus F'$ as a direct sum of two closed subspaces such that the norm on $E$ is
    equivalent to the sum of the norms on $F$ and $F'$-components.

  2. But the first point also raises the suspicion that the question may be a bit ill-defined (and uninteresting) in the sense that the answer might depend on the choice of the norm $leftVertcdotrightVert_E$ among the set of equivalent norms. However, if we, for example, perturb the sup-norm of $C([0,1])$ in the above example by multiplying the functions with a fixed positive definite function before taking the sup-norm, the argument seems to survive, so may be replacing the norm with an equivalent one is irrelevant?


So to satisfy my curiosity I also welcome "your favorite example" (one with a finite-dimensional $F$ would be nice to see), where we absolutely need the infimum here. Bits about any sufficient or necessary conditions for the minimum to be sufficient or (as a last resort :-) pointers to relevant literature are, of course, also appreciated.










share|cite|improve this question











$endgroup$








  • 2




    $begingroup$
    I seem to recall that the minimum is attained in any reflexive Banach space, but I may be mistaken.
    $endgroup$
    – JSchlather
    Feb 10 '13 at 21:58






  • 1




    $begingroup$
    Thanks, @Jacob ! The Wikipedia article on reflexive spaces seems to support that (the distance from a point to a closed convex set attains its infimum).
    $endgroup$
    – Jyrki Lahtonen
    Feb 10 '13 at 22:07






  • 2




    $begingroup$
    I vaguely recall this has to do with the proximinality (or proximality, not sure) of the subspace. But I can't find any good reference.
    $endgroup$
    – Julien
    Feb 10 '13 at 22:11






  • 3




    $begingroup$
    See this post. The link in my comment there gives a general method for constructing a counterexample in a nonreflexive space, as well as a concrete example for $ell_1$.
    $endgroup$
    – David Mitra
    Feb 10 '13 at 22:48








  • 1




    $begingroup$
    Example 71 of these notes gives a coherent proof that in any reflexive space, the distance from a point to a closed, convex, non-empty set is attained. Of course, from the other link I mentioned, it follows that the converse holds.
    $endgroup$
    – David Mitra
    Feb 10 '13 at 23:03
















38












$begingroup$


Let $E$ be a Banach space and $F$ a closed subspace. It is well known that the quotient space $E/F$ is also a Banach space with respect to the norm
$$
leftVert x+FrightVert_{E/F}=inf{leftVert yrightVert_Emid yin x+F}.
$$
Unfortunately in a set of lecture notes on (Lie) group representations (material for our study group) the author accidentally used here $min$ instead of $inf$. Probably a mostly harmless booboo, because at that point it was only needed to get a Banach space structure on the quotient, and we will probably be concentrating on Hilbert spaces anyway, where the problem does not arise.



Namely from Rudin's Functional Analysis I could not find a proof that the minimum should always be attained. Except in the case of a Hilbert space, where an application of parallelogram law (the sum of the squared norms of the two diagonals of a parallelogram equals that of the four sides) allows us to find a Cauchy sequence among a sequence of vectors $(y_n)subset x+F$ such that
$$lim_{ntoinfty}leftVert y_nrightVert_E=leftVert x+FrightVert_{E/F}.$$



But anyway, the suspicion was left that the infimum is there for a reason (other than conveniently allowing us to sweep this detail under the rug at that point of the development of theory), so in the interest of serving our study group I had to come up with a specific example, where the minimum is not achieved. It's been 25 years since I really had to exercise the Banach space gland in my brain, so it has shrunk to size of a raisin. Searching this site did help, because I found this question. There we have $E=C([0,1])$, the space of continuous real functions on $[0,1]$ equipped with the sup-norm. If we denote by $Lambda$ the continuous functional
$$
Lambda: Etomathbb{R},fmapstoint_0^{1/2}f-int_{1/2}^1f
$$
and let $F=kerLambda$, then the answer to the linked question proves that there is no minimum sup-norm function in the coset $Lambda^{-1}(1)$.



So I have a (counter)example, and the main question has evolved to:




When can we use minimum in place of infimum in the definition of the quotient space norm?




My thinking:




  1. It seems to me that the answer is affirmative, if $F$ has a complement, i.e. we can write $E=Foplus F'$ as a direct sum of two closed subspaces such that the norm on $E$ is
    equivalent to the sum of the norms on $F$ and $F'$-components.

  2. But the first point also raises the suspicion that the question may be a bit ill-defined (and uninteresting) in the sense that the answer might depend on the choice of the norm $leftVertcdotrightVert_E$ among the set of equivalent norms. However, if we, for example, perturb the sup-norm of $C([0,1])$ in the above example by multiplying the functions with a fixed positive definite function before taking the sup-norm, the argument seems to survive, so may be replacing the norm with an equivalent one is irrelevant?


So to satisfy my curiosity I also welcome "your favorite example" (one with a finite-dimensional $F$ would be nice to see), where we absolutely need the infimum here. Bits about any sufficient or necessary conditions for the minimum to be sufficient or (as a last resort :-) pointers to relevant literature are, of course, also appreciated.










share|cite|improve this question











$endgroup$








  • 2




    $begingroup$
    I seem to recall that the minimum is attained in any reflexive Banach space, but I may be mistaken.
    $endgroup$
    – JSchlather
    Feb 10 '13 at 21:58






  • 1




    $begingroup$
    Thanks, @Jacob ! The Wikipedia article on reflexive spaces seems to support that (the distance from a point to a closed convex set attains its infimum).
    $endgroup$
    – Jyrki Lahtonen
    Feb 10 '13 at 22:07






  • 2




    $begingroup$
    I vaguely recall this has to do with the proximinality (or proximality, not sure) of the subspace. But I can't find any good reference.
    $endgroup$
    – Julien
    Feb 10 '13 at 22:11






  • 3




    $begingroup$
    See this post. The link in my comment there gives a general method for constructing a counterexample in a nonreflexive space, as well as a concrete example for $ell_1$.
    $endgroup$
    – David Mitra
    Feb 10 '13 at 22:48








  • 1




    $begingroup$
    Example 71 of these notes gives a coherent proof that in any reflexive space, the distance from a point to a closed, convex, non-empty set is attained. Of course, from the other link I mentioned, it follows that the converse holds.
    $endgroup$
    – David Mitra
    Feb 10 '13 at 23:03














38












38








38


15



$begingroup$


Let $E$ be a Banach space and $F$ a closed subspace. It is well known that the quotient space $E/F$ is also a Banach space with respect to the norm
$$
leftVert x+FrightVert_{E/F}=inf{leftVert yrightVert_Emid yin x+F}.
$$
Unfortunately in a set of lecture notes on (Lie) group representations (material for our study group) the author accidentally used here $min$ instead of $inf$. Probably a mostly harmless booboo, because at that point it was only needed to get a Banach space structure on the quotient, and we will probably be concentrating on Hilbert spaces anyway, where the problem does not arise.



Namely from Rudin's Functional Analysis I could not find a proof that the minimum should always be attained. Except in the case of a Hilbert space, where an application of parallelogram law (the sum of the squared norms of the two diagonals of a parallelogram equals that of the four sides) allows us to find a Cauchy sequence among a sequence of vectors $(y_n)subset x+F$ such that
$$lim_{ntoinfty}leftVert y_nrightVert_E=leftVert x+FrightVert_{E/F}.$$



But anyway, the suspicion was left that the infimum is there for a reason (other than conveniently allowing us to sweep this detail under the rug at that point of the development of theory), so in the interest of serving our study group I had to come up with a specific example, where the minimum is not achieved. It's been 25 years since I really had to exercise the Banach space gland in my brain, so it has shrunk to size of a raisin. Searching this site did help, because I found this question. There we have $E=C([0,1])$, the space of continuous real functions on $[0,1]$ equipped with the sup-norm. If we denote by $Lambda$ the continuous functional
$$
Lambda: Etomathbb{R},fmapstoint_0^{1/2}f-int_{1/2}^1f
$$
and let $F=kerLambda$, then the answer to the linked question proves that there is no minimum sup-norm function in the coset $Lambda^{-1}(1)$.



So I have a (counter)example, and the main question has evolved to:




When can we use minimum in place of infimum in the definition of the quotient space norm?




My thinking:




  1. It seems to me that the answer is affirmative, if $F$ has a complement, i.e. we can write $E=Foplus F'$ as a direct sum of two closed subspaces such that the norm on $E$ is
    equivalent to the sum of the norms on $F$ and $F'$-components.

  2. But the first point also raises the suspicion that the question may be a bit ill-defined (and uninteresting) in the sense that the answer might depend on the choice of the norm $leftVertcdotrightVert_E$ among the set of equivalent norms. However, if we, for example, perturb the sup-norm of $C([0,1])$ in the above example by multiplying the functions with a fixed positive definite function before taking the sup-norm, the argument seems to survive, so may be replacing the norm with an equivalent one is irrelevant?


So to satisfy my curiosity I also welcome "your favorite example" (one with a finite-dimensional $F$ would be nice to see), where we absolutely need the infimum here. Bits about any sufficient or necessary conditions for the minimum to be sufficient or (as a last resort :-) pointers to relevant literature are, of course, also appreciated.










share|cite|improve this question











$endgroup$




Let $E$ be a Banach space and $F$ a closed subspace. It is well known that the quotient space $E/F$ is also a Banach space with respect to the norm
$$
leftVert x+FrightVert_{E/F}=inf{leftVert yrightVert_Emid yin x+F}.
$$
Unfortunately in a set of lecture notes on (Lie) group representations (material for our study group) the author accidentally used here $min$ instead of $inf$. Probably a mostly harmless booboo, because at that point it was only needed to get a Banach space structure on the quotient, and we will probably be concentrating on Hilbert spaces anyway, where the problem does not arise.



Namely from Rudin's Functional Analysis I could not find a proof that the minimum should always be attained. Except in the case of a Hilbert space, where an application of parallelogram law (the sum of the squared norms of the two diagonals of a parallelogram equals that of the four sides) allows us to find a Cauchy sequence among a sequence of vectors $(y_n)subset x+F$ such that
$$lim_{ntoinfty}leftVert y_nrightVert_E=leftVert x+FrightVert_{E/F}.$$



But anyway, the suspicion was left that the infimum is there for a reason (other than conveniently allowing us to sweep this detail under the rug at that point of the development of theory), so in the interest of serving our study group I had to come up with a specific example, where the minimum is not achieved. It's been 25 years since I really had to exercise the Banach space gland in my brain, so it has shrunk to size of a raisin. Searching this site did help, because I found this question. There we have $E=C([0,1])$, the space of continuous real functions on $[0,1]$ equipped with the sup-norm. If we denote by $Lambda$ the continuous functional
$$
Lambda: Etomathbb{R},fmapstoint_0^{1/2}f-int_{1/2}^1f
$$
and let $F=kerLambda$, then the answer to the linked question proves that there is no minimum sup-norm function in the coset $Lambda^{-1}(1)$.



So I have a (counter)example, and the main question has evolved to:




When can we use minimum in place of infimum in the definition of the quotient space norm?




My thinking:




  1. It seems to me that the answer is affirmative, if $F$ has a complement, i.e. we can write $E=Foplus F'$ as a direct sum of two closed subspaces such that the norm on $E$ is
    equivalent to the sum of the norms on $F$ and $F'$-components.

  2. But the first point also raises the suspicion that the question may be a bit ill-defined (and uninteresting) in the sense that the answer might depend on the choice of the norm $leftVertcdotrightVert_E$ among the set of equivalent norms. However, if we, for example, perturb the sup-norm of $C([0,1])$ in the above example by multiplying the functions with a fixed positive definite function before taking the sup-norm, the argument seems to survive, so may be replacing the norm with an equivalent one is irrelevant?


So to satisfy my curiosity I also welcome "your favorite example" (one with a finite-dimensional $F$ would be nice to see), where we absolutely need the infimum here. Bits about any sufficient or necessary conditions for the minimum to be sufficient or (as a last resort :-) pointers to relevant literature are, of course, also appreciated.







functional-analysis banach-spaces norm quotient-spaces






share|cite|improve this question















share|cite|improve this question













share|cite|improve this question




share|cite|improve this question








edited Dec 8 '18 at 11:43









Martin Sleziak

44.7k9117272




44.7k9117272










asked Feb 10 '13 at 21:35









Jyrki LahtonenJyrki Lahtonen

109k13169372




109k13169372








  • 2




    $begingroup$
    I seem to recall that the minimum is attained in any reflexive Banach space, but I may be mistaken.
    $endgroup$
    – JSchlather
    Feb 10 '13 at 21:58






  • 1




    $begingroup$
    Thanks, @Jacob ! The Wikipedia article on reflexive spaces seems to support that (the distance from a point to a closed convex set attains its infimum).
    $endgroup$
    – Jyrki Lahtonen
    Feb 10 '13 at 22:07






  • 2




    $begingroup$
    I vaguely recall this has to do with the proximinality (or proximality, not sure) of the subspace. But I can't find any good reference.
    $endgroup$
    – Julien
    Feb 10 '13 at 22:11






  • 3




    $begingroup$
    See this post. The link in my comment there gives a general method for constructing a counterexample in a nonreflexive space, as well as a concrete example for $ell_1$.
    $endgroup$
    – David Mitra
    Feb 10 '13 at 22:48








  • 1




    $begingroup$
    Example 71 of these notes gives a coherent proof that in any reflexive space, the distance from a point to a closed, convex, non-empty set is attained. Of course, from the other link I mentioned, it follows that the converse holds.
    $endgroup$
    – David Mitra
    Feb 10 '13 at 23:03














  • 2




    $begingroup$
    I seem to recall that the minimum is attained in any reflexive Banach space, but I may be mistaken.
    $endgroup$
    – JSchlather
    Feb 10 '13 at 21:58






  • 1




    $begingroup$
    Thanks, @Jacob ! The Wikipedia article on reflexive spaces seems to support that (the distance from a point to a closed convex set attains its infimum).
    $endgroup$
    – Jyrki Lahtonen
    Feb 10 '13 at 22:07






  • 2




    $begingroup$
    I vaguely recall this has to do with the proximinality (or proximality, not sure) of the subspace. But I can't find any good reference.
    $endgroup$
    – Julien
    Feb 10 '13 at 22:11






  • 3




    $begingroup$
    See this post. The link in my comment there gives a general method for constructing a counterexample in a nonreflexive space, as well as a concrete example for $ell_1$.
    $endgroup$
    – David Mitra
    Feb 10 '13 at 22:48








  • 1




    $begingroup$
    Example 71 of these notes gives a coherent proof that in any reflexive space, the distance from a point to a closed, convex, non-empty set is attained. Of course, from the other link I mentioned, it follows that the converse holds.
    $endgroup$
    – David Mitra
    Feb 10 '13 at 23:03








2




2




$begingroup$
I seem to recall that the minimum is attained in any reflexive Banach space, but I may be mistaken.
$endgroup$
– JSchlather
Feb 10 '13 at 21:58




$begingroup$
I seem to recall that the minimum is attained in any reflexive Banach space, but I may be mistaken.
$endgroup$
– JSchlather
Feb 10 '13 at 21:58




1




1




$begingroup$
Thanks, @Jacob ! The Wikipedia article on reflexive spaces seems to support that (the distance from a point to a closed convex set attains its infimum).
$endgroup$
– Jyrki Lahtonen
Feb 10 '13 at 22:07




$begingroup$
Thanks, @Jacob ! The Wikipedia article on reflexive spaces seems to support that (the distance from a point to a closed convex set attains its infimum).
$endgroup$
– Jyrki Lahtonen
Feb 10 '13 at 22:07




2




2




$begingroup$
I vaguely recall this has to do with the proximinality (or proximality, not sure) of the subspace. But I can't find any good reference.
$endgroup$
– Julien
Feb 10 '13 at 22:11




$begingroup$
I vaguely recall this has to do with the proximinality (or proximality, not sure) of the subspace. But I can't find any good reference.
$endgroup$
– Julien
Feb 10 '13 at 22:11




3




3




$begingroup$
See this post. The link in my comment there gives a general method for constructing a counterexample in a nonreflexive space, as well as a concrete example for $ell_1$.
$endgroup$
– David Mitra
Feb 10 '13 at 22:48






$begingroup$
See this post. The link in my comment there gives a general method for constructing a counterexample in a nonreflexive space, as well as a concrete example for $ell_1$.
$endgroup$
– David Mitra
Feb 10 '13 at 22:48






1




1




$begingroup$
Example 71 of these notes gives a coherent proof that in any reflexive space, the distance from a point to a closed, convex, non-empty set is attained. Of course, from the other link I mentioned, it follows that the converse holds.
$endgroup$
– David Mitra
Feb 10 '13 at 23:03




$begingroup$
Example 71 of these notes gives a coherent proof that in any reflexive space, the distance from a point to a closed, convex, non-empty set is attained. Of course, from the other link I mentioned, it follows that the converse holds.
$endgroup$
– David Mitra
Feb 10 '13 at 23:03










1 Answer
1






active

oldest

votes


















26












$begingroup$

James's theorem asserts the following: if every continuous linear functional on $E$ attains its norm then $E$ is reflexive. This means that on every non-reflexive space we can find a codimension 1 subspace for which the quotient norm needs the infimum in place of a minimum.



In the comments there were already a few links to threads discussing explicit examples, let me list them for the sake of completeness:




  • Given a point $x$ and a closed subspace $Y$ of a normed space, must the distance from $x$ to $Y$ be achieved by some $yin Y$? where David Mitra mentions this blog post with a nice discussion.


  • Distance minimizers in $L^1$ and $L^{infty}$ contains more examples and links.





You asked for a finite-dimensional example. This is impossible: for if $F$ is a finite-dimensional subspace of $E$ then we can use compactness of closed and bounded sets in $F$ to ascertain that the infimum is in fact a minimum.



Let $x in E$ be arbitrary. Choose a sequence $y_{n} in F$ such that $lVert x - y_nrVert to lVert xrVert_{E/F}$. By the reverse triangle inequality $lVert y_n - xrVert geq lvert lVert y_nrVert - lVert xrVertrvert$ we see that $lVert y_nrVert leq 3lVert xrVert$ for large enough $n$, so the sequence $(y_n)$ is contained in a bounded subset of $F$. Pass to a convergent subsequence $y_{n_i} to f in F$ of $(y_n)$ and observe that $$lVert xrVert_{E/F} = lim lVert x - y_{n_i}rVert = lVert x - frVert.$$





With a little more work and refining the above idea, one can prove that a sufficient condition is reflexivity of the subspace $F$.



To see this, recall the following facts:





  1. A norm-closed convex subset $C$ of a Banach space is weakly closed.



    By the Hahn-Banach separation theorem we can write $C$ as the intersection of closed half-spaces defined by linear functionals, and these half-spaces are weakly closed since the weak topology is the initial topology induced by the linear functionals.




  2. The closed unit ball of a reflexive Banach space is weakly compact (and hence every closed ball is weakly compact).



    Since $E$ is reflexive, it is the dual space of $E^{ast}$ and the weak and weak*-topologies on $E$ coincide. Since the closed unit ball in a dual space is weak*-compact by Alaoglu's theorem, it is therefore weakly compact.



  3. Combining 1. and 2. we see that every closed and bounded convex set in a reflexive space is weakly compact. [One can also prove that a closed and bounded convex set in a reflexive space is weakly sequentially compact but we won't need this here].



  4. The norm on a Banach space is weakly lower semicontinuous: if a net $x_i$ converges weakly to $x$ then $lVert x rVert leq liminf_i lVert x_irVert$.



    By Hahn-Banach there is a norm 1-functional $varphi in E^ast$ such that $varphi(x) = lVert xrVert$. But then $lVert xrVert = lim_{i} lvert varphi(x_i)rvert leq liminf_i lVert varphirVertlVert x_irVert = liminf_i lVert x_irVert$.




  5. The restriction of weak topology on $E$ to a closed subspace $F$ of $E$ is the same as the weak topology of $F$ as a Banach space.



    This follows from Hahn-Banach.




We are finally ready to prove the announced result:




Let $F$ be a reflexive subspace of the Banach space $E$. Let $x in E$ be arbitrary. Then there is $f in F$ such that $lVert xrVert_{E/F} = lVert x - frVert = inf_{y in F} lVert x - yrVert$.




Choose a sequence $y_{n} in F$ such that $lVert x - y_nrVert to lVert xrVert_{E/F}$. Again the reverse triangle inequality shows that $lVert y_nrVert leq 3lVert xrVert$ for large enough $n$, so the sequence $(y_n)$ is contained in a weakly compact convex subset of $F$. Pass to a weakly convergent subnet $y_{n_i} to f in F$ of $(y_n)$ and observe that $$lVert xrVert_{E/F} leq lVert x -frVert leq liminf lVert x - y_{n_i}rVert = liminf lVert x - y_nrVert = lVert xrVert_{E/F}.$$






share|cite|improve this answer











$endgroup$









  • 1




    $begingroup$
    Thanks, Martin. This argument should be accessible to me. I will need to find the time to absorb the details before I accept, but looks good.
    $endgroup$
    – Jyrki Lahtonen
    Feb 11 '13 at 6:17










  • $begingroup$
    Thanks. Somehow I'm unhappy with the longish and clumsy argument since the facts I outline seem a bit redundant. The basic idea is to find a substitute for norm compactness yielding the minimizer in the finite-dimensional case and weak compactness is the only one I could think of. Probably there are more efficient and clearer ways to put it. Basically the two ideas are: 1) the function $y mapsto lVert x-yrVert$ attains its minimum on every weakly compact convex subset. 2) a minimizing sequence in the reflexive subspace $F$ must be contained in a weakly compact convex subset of $F$.
    $endgroup$
    – Martin
    Feb 11 '13 at 10:02











Your Answer





StackExchange.ifUsing("editor", function () {
return StackExchange.using("mathjaxEditing", function () {
StackExchange.MarkdownEditor.creationCallbacks.add(function (editor, postfix) {
StackExchange.mathjaxEditing.prepareWmdForMathJax(editor, postfix, [["$", "$"], ["\\(","\\)"]]);
});
});
}, "mathjax-editing");

StackExchange.ready(function() {
var channelOptions = {
tags: "".split(" "),
id: "69"
};
initTagRenderer("".split(" "), "".split(" "), channelOptions);

StackExchange.using("externalEditor", function() {
// Have to fire editor after snippets, if snippets enabled
if (StackExchange.settings.snippets.snippetsEnabled) {
StackExchange.using("snippets", function() {
createEditor();
});
}
else {
createEditor();
}
});

function createEditor() {
StackExchange.prepareEditor({
heartbeatType: 'answer',
autoActivateHeartbeat: false,
convertImagesToLinks: true,
noModals: true,
showLowRepImageUploadWarning: true,
reputationToPostImages: 10,
bindNavPrevention: true,
postfix: "",
imageUploader: {
brandingHtml: "Powered by u003ca class="icon-imgur-white" href="https://imgur.com/"u003eu003c/au003e",
contentPolicyHtml: "User contributions licensed under u003ca href="https://creativecommons.org/licenses/by-sa/3.0/"u003ecc by-sa 3.0 with attribution requiredu003c/au003e u003ca href="https://stackoverflow.com/legal/content-policy"u003e(content policy)u003c/au003e",
allowUrls: true
},
noCode: true, onDemand: true,
discardSelector: ".discard-answer"
,immediatelyShowMarkdownHelp:true
});


}
});














draft saved

draft discarded


















StackExchange.ready(
function () {
StackExchange.openid.initPostLogin('.new-post-login', 'https%3a%2f%2fmath.stackexchange.com%2fquestions%2f299735%2fon-the-norm-of-a-quotient-of-a-banach-space%23new-answer', 'question_page');
}
);

Post as a guest















Required, but never shown

























1 Answer
1






active

oldest

votes








1 Answer
1






active

oldest

votes









active

oldest

votes






active

oldest

votes









26












$begingroup$

James's theorem asserts the following: if every continuous linear functional on $E$ attains its norm then $E$ is reflexive. This means that on every non-reflexive space we can find a codimension 1 subspace for which the quotient norm needs the infimum in place of a minimum.



In the comments there were already a few links to threads discussing explicit examples, let me list them for the sake of completeness:




  • Given a point $x$ and a closed subspace $Y$ of a normed space, must the distance from $x$ to $Y$ be achieved by some $yin Y$? where David Mitra mentions this blog post with a nice discussion.


  • Distance minimizers in $L^1$ and $L^{infty}$ contains more examples and links.





You asked for a finite-dimensional example. This is impossible: for if $F$ is a finite-dimensional subspace of $E$ then we can use compactness of closed and bounded sets in $F$ to ascertain that the infimum is in fact a minimum.



Let $x in E$ be arbitrary. Choose a sequence $y_{n} in F$ such that $lVert x - y_nrVert to lVert xrVert_{E/F}$. By the reverse triangle inequality $lVert y_n - xrVert geq lvert lVert y_nrVert - lVert xrVertrvert$ we see that $lVert y_nrVert leq 3lVert xrVert$ for large enough $n$, so the sequence $(y_n)$ is contained in a bounded subset of $F$. Pass to a convergent subsequence $y_{n_i} to f in F$ of $(y_n)$ and observe that $$lVert xrVert_{E/F} = lim lVert x - y_{n_i}rVert = lVert x - frVert.$$





With a little more work and refining the above idea, one can prove that a sufficient condition is reflexivity of the subspace $F$.



To see this, recall the following facts:





  1. A norm-closed convex subset $C$ of a Banach space is weakly closed.



    By the Hahn-Banach separation theorem we can write $C$ as the intersection of closed half-spaces defined by linear functionals, and these half-spaces are weakly closed since the weak topology is the initial topology induced by the linear functionals.




  2. The closed unit ball of a reflexive Banach space is weakly compact (and hence every closed ball is weakly compact).



    Since $E$ is reflexive, it is the dual space of $E^{ast}$ and the weak and weak*-topologies on $E$ coincide. Since the closed unit ball in a dual space is weak*-compact by Alaoglu's theorem, it is therefore weakly compact.



  3. Combining 1. and 2. we see that every closed and bounded convex set in a reflexive space is weakly compact. [One can also prove that a closed and bounded convex set in a reflexive space is weakly sequentially compact but we won't need this here].



  4. The norm on a Banach space is weakly lower semicontinuous: if a net $x_i$ converges weakly to $x$ then $lVert x rVert leq liminf_i lVert x_irVert$.



    By Hahn-Banach there is a norm 1-functional $varphi in E^ast$ such that $varphi(x) = lVert xrVert$. But then $lVert xrVert = lim_{i} lvert varphi(x_i)rvert leq liminf_i lVert varphirVertlVert x_irVert = liminf_i lVert x_irVert$.




  5. The restriction of weak topology on $E$ to a closed subspace $F$ of $E$ is the same as the weak topology of $F$ as a Banach space.



    This follows from Hahn-Banach.




We are finally ready to prove the announced result:




Let $F$ be a reflexive subspace of the Banach space $E$. Let $x in E$ be arbitrary. Then there is $f in F$ such that $lVert xrVert_{E/F} = lVert x - frVert = inf_{y in F} lVert x - yrVert$.




Choose a sequence $y_{n} in F$ such that $lVert x - y_nrVert to lVert xrVert_{E/F}$. Again the reverse triangle inequality shows that $lVert y_nrVert leq 3lVert xrVert$ for large enough $n$, so the sequence $(y_n)$ is contained in a weakly compact convex subset of $F$. Pass to a weakly convergent subnet $y_{n_i} to f in F$ of $(y_n)$ and observe that $$lVert xrVert_{E/F} leq lVert x -frVert leq liminf lVert x - y_{n_i}rVert = liminf lVert x - y_nrVert = lVert xrVert_{E/F}.$$






share|cite|improve this answer











$endgroup$









  • 1




    $begingroup$
    Thanks, Martin. This argument should be accessible to me. I will need to find the time to absorb the details before I accept, but looks good.
    $endgroup$
    – Jyrki Lahtonen
    Feb 11 '13 at 6:17










  • $begingroup$
    Thanks. Somehow I'm unhappy with the longish and clumsy argument since the facts I outline seem a bit redundant. The basic idea is to find a substitute for norm compactness yielding the minimizer in the finite-dimensional case and weak compactness is the only one I could think of. Probably there are more efficient and clearer ways to put it. Basically the two ideas are: 1) the function $y mapsto lVert x-yrVert$ attains its minimum on every weakly compact convex subset. 2) a minimizing sequence in the reflexive subspace $F$ must be contained in a weakly compact convex subset of $F$.
    $endgroup$
    – Martin
    Feb 11 '13 at 10:02
















26












$begingroup$

James's theorem asserts the following: if every continuous linear functional on $E$ attains its norm then $E$ is reflexive. This means that on every non-reflexive space we can find a codimension 1 subspace for which the quotient norm needs the infimum in place of a minimum.



In the comments there were already a few links to threads discussing explicit examples, let me list them for the sake of completeness:




  • Given a point $x$ and a closed subspace $Y$ of a normed space, must the distance from $x$ to $Y$ be achieved by some $yin Y$? where David Mitra mentions this blog post with a nice discussion.


  • Distance minimizers in $L^1$ and $L^{infty}$ contains more examples and links.





You asked for a finite-dimensional example. This is impossible: for if $F$ is a finite-dimensional subspace of $E$ then we can use compactness of closed and bounded sets in $F$ to ascertain that the infimum is in fact a minimum.



Let $x in E$ be arbitrary. Choose a sequence $y_{n} in F$ such that $lVert x - y_nrVert to lVert xrVert_{E/F}$. By the reverse triangle inequality $lVert y_n - xrVert geq lvert lVert y_nrVert - lVert xrVertrvert$ we see that $lVert y_nrVert leq 3lVert xrVert$ for large enough $n$, so the sequence $(y_n)$ is contained in a bounded subset of $F$. Pass to a convergent subsequence $y_{n_i} to f in F$ of $(y_n)$ and observe that $$lVert xrVert_{E/F} = lim lVert x - y_{n_i}rVert = lVert x - frVert.$$





With a little more work and refining the above idea, one can prove that a sufficient condition is reflexivity of the subspace $F$.



To see this, recall the following facts:





  1. A norm-closed convex subset $C$ of a Banach space is weakly closed.



    By the Hahn-Banach separation theorem we can write $C$ as the intersection of closed half-spaces defined by linear functionals, and these half-spaces are weakly closed since the weak topology is the initial topology induced by the linear functionals.




  2. The closed unit ball of a reflexive Banach space is weakly compact (and hence every closed ball is weakly compact).



    Since $E$ is reflexive, it is the dual space of $E^{ast}$ and the weak and weak*-topologies on $E$ coincide. Since the closed unit ball in a dual space is weak*-compact by Alaoglu's theorem, it is therefore weakly compact.



  3. Combining 1. and 2. we see that every closed and bounded convex set in a reflexive space is weakly compact. [One can also prove that a closed and bounded convex set in a reflexive space is weakly sequentially compact but we won't need this here].



  4. The norm on a Banach space is weakly lower semicontinuous: if a net $x_i$ converges weakly to $x$ then $lVert x rVert leq liminf_i lVert x_irVert$.



    By Hahn-Banach there is a norm 1-functional $varphi in E^ast$ such that $varphi(x) = lVert xrVert$. But then $lVert xrVert = lim_{i} lvert varphi(x_i)rvert leq liminf_i lVert varphirVertlVert x_irVert = liminf_i lVert x_irVert$.




  5. The restriction of weak topology on $E$ to a closed subspace $F$ of $E$ is the same as the weak topology of $F$ as a Banach space.



    This follows from Hahn-Banach.




We are finally ready to prove the announced result:




Let $F$ be a reflexive subspace of the Banach space $E$. Let $x in E$ be arbitrary. Then there is $f in F$ such that $lVert xrVert_{E/F} = lVert x - frVert = inf_{y in F} lVert x - yrVert$.




Choose a sequence $y_{n} in F$ such that $lVert x - y_nrVert to lVert xrVert_{E/F}$. Again the reverse triangle inequality shows that $lVert y_nrVert leq 3lVert xrVert$ for large enough $n$, so the sequence $(y_n)$ is contained in a weakly compact convex subset of $F$. Pass to a weakly convergent subnet $y_{n_i} to f in F$ of $(y_n)$ and observe that $$lVert xrVert_{E/F} leq lVert x -frVert leq liminf lVert x - y_{n_i}rVert = liminf lVert x - y_nrVert = lVert xrVert_{E/F}.$$






share|cite|improve this answer











$endgroup$









  • 1




    $begingroup$
    Thanks, Martin. This argument should be accessible to me. I will need to find the time to absorb the details before I accept, but looks good.
    $endgroup$
    – Jyrki Lahtonen
    Feb 11 '13 at 6:17










  • $begingroup$
    Thanks. Somehow I'm unhappy with the longish and clumsy argument since the facts I outline seem a bit redundant. The basic idea is to find a substitute for norm compactness yielding the minimizer in the finite-dimensional case and weak compactness is the only one I could think of. Probably there are more efficient and clearer ways to put it. Basically the two ideas are: 1) the function $y mapsto lVert x-yrVert$ attains its minimum on every weakly compact convex subset. 2) a minimizing sequence in the reflexive subspace $F$ must be contained in a weakly compact convex subset of $F$.
    $endgroup$
    – Martin
    Feb 11 '13 at 10:02














26












26








26





$begingroup$

James's theorem asserts the following: if every continuous linear functional on $E$ attains its norm then $E$ is reflexive. This means that on every non-reflexive space we can find a codimension 1 subspace for which the quotient norm needs the infimum in place of a minimum.



In the comments there were already a few links to threads discussing explicit examples, let me list them for the sake of completeness:




  • Given a point $x$ and a closed subspace $Y$ of a normed space, must the distance from $x$ to $Y$ be achieved by some $yin Y$? where David Mitra mentions this blog post with a nice discussion.


  • Distance minimizers in $L^1$ and $L^{infty}$ contains more examples and links.





You asked for a finite-dimensional example. This is impossible: for if $F$ is a finite-dimensional subspace of $E$ then we can use compactness of closed and bounded sets in $F$ to ascertain that the infimum is in fact a minimum.



Let $x in E$ be arbitrary. Choose a sequence $y_{n} in F$ such that $lVert x - y_nrVert to lVert xrVert_{E/F}$. By the reverse triangle inequality $lVert y_n - xrVert geq lvert lVert y_nrVert - lVert xrVertrvert$ we see that $lVert y_nrVert leq 3lVert xrVert$ for large enough $n$, so the sequence $(y_n)$ is contained in a bounded subset of $F$. Pass to a convergent subsequence $y_{n_i} to f in F$ of $(y_n)$ and observe that $$lVert xrVert_{E/F} = lim lVert x - y_{n_i}rVert = lVert x - frVert.$$





With a little more work and refining the above idea, one can prove that a sufficient condition is reflexivity of the subspace $F$.



To see this, recall the following facts:





  1. A norm-closed convex subset $C$ of a Banach space is weakly closed.



    By the Hahn-Banach separation theorem we can write $C$ as the intersection of closed half-spaces defined by linear functionals, and these half-spaces are weakly closed since the weak topology is the initial topology induced by the linear functionals.




  2. The closed unit ball of a reflexive Banach space is weakly compact (and hence every closed ball is weakly compact).



    Since $E$ is reflexive, it is the dual space of $E^{ast}$ and the weak and weak*-topologies on $E$ coincide. Since the closed unit ball in a dual space is weak*-compact by Alaoglu's theorem, it is therefore weakly compact.



  3. Combining 1. and 2. we see that every closed and bounded convex set in a reflexive space is weakly compact. [One can also prove that a closed and bounded convex set in a reflexive space is weakly sequentially compact but we won't need this here].



  4. The norm on a Banach space is weakly lower semicontinuous: if a net $x_i$ converges weakly to $x$ then $lVert x rVert leq liminf_i lVert x_irVert$.



    By Hahn-Banach there is a norm 1-functional $varphi in E^ast$ such that $varphi(x) = lVert xrVert$. But then $lVert xrVert = lim_{i} lvert varphi(x_i)rvert leq liminf_i lVert varphirVertlVert x_irVert = liminf_i lVert x_irVert$.




  5. The restriction of weak topology on $E$ to a closed subspace $F$ of $E$ is the same as the weak topology of $F$ as a Banach space.



    This follows from Hahn-Banach.




We are finally ready to prove the announced result:




Let $F$ be a reflexive subspace of the Banach space $E$. Let $x in E$ be arbitrary. Then there is $f in F$ such that $lVert xrVert_{E/F} = lVert x - frVert = inf_{y in F} lVert x - yrVert$.




Choose a sequence $y_{n} in F$ such that $lVert x - y_nrVert to lVert xrVert_{E/F}$. Again the reverse triangle inequality shows that $lVert y_nrVert leq 3lVert xrVert$ for large enough $n$, so the sequence $(y_n)$ is contained in a weakly compact convex subset of $F$. Pass to a weakly convergent subnet $y_{n_i} to f in F$ of $(y_n)$ and observe that $$lVert xrVert_{E/F} leq lVert x -frVert leq liminf lVert x - y_{n_i}rVert = liminf lVert x - y_nrVert = lVert xrVert_{E/F}.$$






share|cite|improve this answer











$endgroup$



James's theorem asserts the following: if every continuous linear functional on $E$ attains its norm then $E$ is reflexive. This means that on every non-reflexive space we can find a codimension 1 subspace for which the quotient norm needs the infimum in place of a minimum.



In the comments there were already a few links to threads discussing explicit examples, let me list them for the sake of completeness:




  • Given a point $x$ and a closed subspace $Y$ of a normed space, must the distance from $x$ to $Y$ be achieved by some $yin Y$? where David Mitra mentions this blog post with a nice discussion.


  • Distance minimizers in $L^1$ and $L^{infty}$ contains more examples and links.





You asked for a finite-dimensional example. This is impossible: for if $F$ is a finite-dimensional subspace of $E$ then we can use compactness of closed and bounded sets in $F$ to ascertain that the infimum is in fact a minimum.



Let $x in E$ be arbitrary. Choose a sequence $y_{n} in F$ such that $lVert x - y_nrVert to lVert xrVert_{E/F}$. By the reverse triangle inequality $lVert y_n - xrVert geq lvert lVert y_nrVert - lVert xrVertrvert$ we see that $lVert y_nrVert leq 3lVert xrVert$ for large enough $n$, so the sequence $(y_n)$ is contained in a bounded subset of $F$. Pass to a convergent subsequence $y_{n_i} to f in F$ of $(y_n)$ and observe that $$lVert xrVert_{E/F} = lim lVert x - y_{n_i}rVert = lVert x - frVert.$$





With a little more work and refining the above idea, one can prove that a sufficient condition is reflexivity of the subspace $F$.



To see this, recall the following facts:





  1. A norm-closed convex subset $C$ of a Banach space is weakly closed.



    By the Hahn-Banach separation theorem we can write $C$ as the intersection of closed half-spaces defined by linear functionals, and these half-spaces are weakly closed since the weak topology is the initial topology induced by the linear functionals.




  2. The closed unit ball of a reflexive Banach space is weakly compact (and hence every closed ball is weakly compact).



    Since $E$ is reflexive, it is the dual space of $E^{ast}$ and the weak and weak*-topologies on $E$ coincide. Since the closed unit ball in a dual space is weak*-compact by Alaoglu's theorem, it is therefore weakly compact.



  3. Combining 1. and 2. we see that every closed and bounded convex set in a reflexive space is weakly compact. [One can also prove that a closed and bounded convex set in a reflexive space is weakly sequentially compact but we won't need this here].



  4. The norm on a Banach space is weakly lower semicontinuous: if a net $x_i$ converges weakly to $x$ then $lVert x rVert leq liminf_i lVert x_irVert$.



    By Hahn-Banach there is a norm 1-functional $varphi in E^ast$ such that $varphi(x) = lVert xrVert$. But then $lVert xrVert = lim_{i} lvert varphi(x_i)rvert leq liminf_i lVert varphirVertlVert x_irVert = liminf_i lVert x_irVert$.




  5. The restriction of weak topology on $E$ to a closed subspace $F$ of $E$ is the same as the weak topology of $F$ as a Banach space.



    This follows from Hahn-Banach.




We are finally ready to prove the announced result:




Let $F$ be a reflexive subspace of the Banach space $E$. Let $x in E$ be arbitrary. Then there is $f in F$ such that $lVert xrVert_{E/F} = lVert x - frVert = inf_{y in F} lVert x - yrVert$.




Choose a sequence $y_{n} in F$ such that $lVert x - y_nrVert to lVert xrVert_{E/F}$. Again the reverse triangle inequality shows that $lVert y_nrVert leq 3lVert xrVert$ for large enough $n$, so the sequence $(y_n)$ is contained in a weakly compact convex subset of $F$. Pass to a weakly convergent subnet $y_{n_i} to f in F$ of $(y_n)$ and observe that $$lVert xrVert_{E/F} leq lVert x -frVert leq liminf lVert x - y_{n_i}rVert = liminf lVert x - y_nrVert = lVert xrVert_{E/F}.$$







share|cite|improve this answer














share|cite|improve this answer



share|cite|improve this answer








edited Apr 13 '17 at 12:20









Community

1




1










answered Feb 11 '13 at 1:35









MartinMartin

6,8922145




6,8922145








  • 1




    $begingroup$
    Thanks, Martin. This argument should be accessible to me. I will need to find the time to absorb the details before I accept, but looks good.
    $endgroup$
    – Jyrki Lahtonen
    Feb 11 '13 at 6:17










  • $begingroup$
    Thanks. Somehow I'm unhappy with the longish and clumsy argument since the facts I outline seem a bit redundant. The basic idea is to find a substitute for norm compactness yielding the minimizer in the finite-dimensional case and weak compactness is the only one I could think of. Probably there are more efficient and clearer ways to put it. Basically the two ideas are: 1) the function $y mapsto lVert x-yrVert$ attains its minimum on every weakly compact convex subset. 2) a minimizing sequence in the reflexive subspace $F$ must be contained in a weakly compact convex subset of $F$.
    $endgroup$
    – Martin
    Feb 11 '13 at 10:02














  • 1




    $begingroup$
    Thanks, Martin. This argument should be accessible to me. I will need to find the time to absorb the details before I accept, but looks good.
    $endgroup$
    – Jyrki Lahtonen
    Feb 11 '13 at 6:17










  • $begingroup$
    Thanks. Somehow I'm unhappy with the longish and clumsy argument since the facts I outline seem a bit redundant. The basic idea is to find a substitute for norm compactness yielding the minimizer in the finite-dimensional case and weak compactness is the only one I could think of. Probably there are more efficient and clearer ways to put it. Basically the two ideas are: 1) the function $y mapsto lVert x-yrVert$ attains its minimum on every weakly compact convex subset. 2) a minimizing sequence in the reflexive subspace $F$ must be contained in a weakly compact convex subset of $F$.
    $endgroup$
    – Martin
    Feb 11 '13 at 10:02








1




1




$begingroup$
Thanks, Martin. This argument should be accessible to me. I will need to find the time to absorb the details before I accept, but looks good.
$endgroup$
– Jyrki Lahtonen
Feb 11 '13 at 6:17




$begingroup$
Thanks, Martin. This argument should be accessible to me. I will need to find the time to absorb the details before I accept, but looks good.
$endgroup$
– Jyrki Lahtonen
Feb 11 '13 at 6:17












$begingroup$
Thanks. Somehow I'm unhappy with the longish and clumsy argument since the facts I outline seem a bit redundant. The basic idea is to find a substitute for norm compactness yielding the minimizer in the finite-dimensional case and weak compactness is the only one I could think of. Probably there are more efficient and clearer ways to put it. Basically the two ideas are: 1) the function $y mapsto lVert x-yrVert$ attains its minimum on every weakly compact convex subset. 2) a minimizing sequence in the reflexive subspace $F$ must be contained in a weakly compact convex subset of $F$.
$endgroup$
– Martin
Feb 11 '13 at 10:02




$begingroup$
Thanks. Somehow I'm unhappy with the longish and clumsy argument since the facts I outline seem a bit redundant. The basic idea is to find a substitute for norm compactness yielding the minimizer in the finite-dimensional case and weak compactness is the only one I could think of. Probably there are more efficient and clearer ways to put it. Basically the two ideas are: 1) the function $y mapsto lVert x-yrVert$ attains its minimum on every weakly compact convex subset. 2) a minimizing sequence in the reflexive subspace $F$ must be contained in a weakly compact convex subset of $F$.
$endgroup$
– Martin
Feb 11 '13 at 10:02


















draft saved

draft discarded




















































Thanks for contributing an answer to Mathematics Stack Exchange!


  • Please be sure to answer the question. Provide details and share your research!

But avoid



  • Asking for help, clarification, or responding to other answers.

  • Making statements based on opinion; back them up with references or personal experience.


Use MathJax to format equations. MathJax reference.


To learn more, see our tips on writing great answers.




draft saved


draft discarded














StackExchange.ready(
function () {
StackExchange.openid.initPostLogin('.new-post-login', 'https%3a%2f%2fmath.stackexchange.com%2fquestions%2f299735%2fon-the-norm-of-a-quotient-of-a-banach-space%23new-answer', 'question_page');
}
);

Post as a guest















Required, but never shown





















































Required, but never shown














Required, but never shown












Required, but never shown







Required, but never shown

































Required, but never shown














Required, but never shown












Required, but never shown







Required, but never shown







Popular posts from this blog

Quarter-circle Tiles

build a pushdown automaton that recognizes the reverse language of a given pushdown automaton?

Mont Emei